• Asignatura: Matemáticas
  • Autor: quispilemajulio5
  • hace 4 años

4m - 3 ⩾2
ayudenme por favor​

Respuestas

Respuesta dada por: CesarGabino
1

Respuesta:

m>5/4

Explicación paso a paso:

Creo que esa es la respuesta

Espero que te sirva

Preguntas similares